Acls Version b Quiz

August 30, 2020 | Author: Anonymous | Category: N/A
Share Embed Donate


Short Description

Download Acls Version b Quiz...

Description

Advanced Cardiac Life Support PRE-TEST 1) Which of the following is true about an oropharyngeal airway? a. It eliminates the need to position the head of the unconscious patient b. It eliminates the possibility of an upper airway obstruction c. It is of no value once a tracheal tube is inserted d. It may stimulate vomiting or laryngospasm if inserted in the semiconscious patient. 2) You have just attempted tracheal intubation. You hear stomach gurgling over the epigastriam, you see no chest expansion, and you are unable to hear breath sounds on either side of the chest during hand ventilation with a bag. Pulse oximetry indicates that the hemoglobin saturation has failed to rise. Which of the following is the most likely explanation of this finding? a. Intubation of the esophagus b. Intubation of the left main bronchus c. Intubation of the right main bronchus d. Unilateral tension pneumothorax 3) Once a tracheal tube is inserted and the position verified (with both primary and secondary confirmation) during CPR, which of the following best describe the ventilations that should be provided? a. An average of 8to 10 ventilations as per minute without pauses for chest compressions. b. Ventilations should provide prompt hyperventilation to correct acidosis, with a pause after every fifth compression c. Ventilations should be delivered with an estimated tidal volume of 3 to 5 mL/kg d. Ventilations should be delivered with room air to avoid hyperoxygenation 4) Which of the following is an indication for tracheal intubation? a. difficulty encountered by qualified rescuers in ventilating an apneic patient with bag-mask device b. a respiratory rate of less than 20 breaths per minutes in a patient with severe chest pain c. the presence of premature ventricular contractions d. to provide airway protection in a responsive patient with an adequate gag reflex

1

5) You are providing hand ventilations with a bag mask for a patient with no spontaneous ventilation. You are explaining the use of bag-mask ventilation to a group of new nurses and residents who are observing your technique. Which of the following statements would most accurately describe the use of bag-mask ventilation during resuscitation? a. bag-mask ventilation can be performed effectively with minimal training and little practice b. bag-mask ventilator will deliver nearly 100% oxygen if a reservoir with a high oxygen flow rate is used c. bag-mask ventilator cannot be performed effectively by one person during resuscitation d. bag-mask ventilator should not be used if the patient makes any spontaneous respiratory effort 6) Which of the following choices lists in correct order the major steps of CPR and AED operation for an unresponsive victim? a. send someone to phone EMS, check for a pulse, attach the AED electrode pads, open the airway, provide 2 breaths if needed, then turn on the AED b. wait for AED and barrier device to arrive, then open the airway, provide 2 breaths if needed, check for a pulse, and if no pulse is present, attach the AED and follow the sequence of AED prompts c. send someone to phone EMS and get the AED, open the airway , provide 2 breaths if needed, check for a pulse, and if no pulse is present, attach the AED and follow the sequence of AED prompts d. provide 2 breaths, check for a pulse, call for the AED, provide chest compressions until the AED arrives, attach the AED 7) In correct order, what are the 4 “universal steps” required to operate an AED? a. POWER ON the AED, attach the AED to the victim, analyze the rhythm, deliver a shock if indicated b. Attach the AED to the victim, POWER ON the AED, analyze the rhythm, deliver a shock if indicated c. Attach the electrode pads to the victim, attach the electronic cables to the AED, POWER ON the AED, analyze the rhythm, deliver a shock if indicated d. POWR ON the AED, attach the AED to the victim, deliver the first shock, analyze the rhythm

2

8-You respond with 2 other rescuers to a 50-year-old man who is unresponsive, pulseless, and not breathing. What tasks would you assign the other rescuers while you set up the AED? a. one rescuer should phone EMS and the other rescuer should begin CPR b. both rescuer should help set up the AED and provide CPR c. one rescuer should open the airway and begin rescue breathing, and the second rescuer should begin chest compressions d. recruit additional first responders to help 9-You attach an AED to a 43-year-old victim who is pulse less and not breathing , and the AED advises “no shock indicated.” What should you do? a. reanalyze immediately b. perform CPR for 2 minute and reanalyze c. perform CPR until EMS personnel arrive d. remove the AED 10-A patient develops sudden VF arrest during evaluation for chest pain in an outpatient clinic. The patient has just received the first shock from the clinic AED. The monitor screen on the AED displays VF. What is the next action rescuers should take? a. resume CPR for approximately 2 minute; then reanalyze the rhythm b. establish IV access for medication administration c. press the ANALYZE control on the AED to reanalyze the rhythm d. administer epinephrine 1 mg IV as soon as the IV is established 11-Which of the following therapies is the most important intervention for VF/Pulseless VT, with the greatest effect on survival to hospital discharge? a. epinephrine b. defibrillator c. oxygen d. amiodarone 12-A 53-year-old man has suffered VF arrest in the ED. He remains in VF after 3 shocks and epinephrine. The team leader asks for amiodarone 300 mg IV. Which of the following statements is true about amiodarone for refractory VF? a. Amiodarone is a Class IIb-recommended therapy for treatment of patients who have not responded to 3 shocks and IV epinephrine b. Amiodarone is associated with better 10year survival than that for other therapy for people who remain in VF after 3 shocks and 1 mg epinephrine Amiodarone is not recommended for refractory VF c.Amiodarone should be administered as soon as an IV is available and at least 1 shock has failed to achieve defibrillation

3

13-A 75-year-old homeless man is in cardiac arrest with pulseless VT at a rate of 220 bpm. After CPR, a DC shock, 1 mg IV epinephrine, then one more shock, the man continues to be polymorphic pulseless VT. He appears wasted and malnourished. The paramedics recognize him as a chronic alcoholic known in the neighborhood. Since he remains in VT after 6 shocks, you are considering an antiarrhythmic. Which of the following agents would be most appropriate for this patient at this time? a. amiodarone c. magnesium b. diltiazem 14Which statement is true about the use of antiarrhythmics for patients with shock-refractory VF/Pulseless VT? A .antiarrhythmic agents are indicated because of the well-documented, long-term benefits of increasing 1-year survival for VF/VT victims B .antiarrhythmic agents can replace the need for continued shocks if infused fast and early c. procainamide antiarrhythmic agents reduce myocardial damage from continued shocks d. in prospective, randomized trials, antiarrhythmic agents have not yet been found to improve survival to hospital discharge of patients with VF/Pulseless VT 15-you are called to assist in the attempted resuscitation of a patient who is demonstrating PEA. As you hurry to the patient’s room, you review the information you learned in the ACLS course about management of PEA. Which one of the following statements about PEA is true? a. chest compressions should be administered only if patient with PEA develops a ventricular rate of less than 50 bpm b.successful treatment of PEA requires identification and treatment of reversible causes, such as the 5 H’s and 5 T’s c.atropine is the drug of choice for treatment of PEA, whether the ventricular rate is slow or fast d.PEA is rarely caused by hypovolemia, so fluid administration is contraindicated and should not be attempted 16 -you are participating in the resuscitation of a patient with PEA. The patient has been intubated (with tube position confirmed) and is receiving 100% oxygen and effective ventilation with bilateral breath sounds and good expansion. Epinephrine 1 mg was administered 2 minutes ago. PEA continues, with ventricular rate of 45 bpm/ While you search for reversible causes of the PEA, which of the following therapies would now be appropriate? a.monophasic defibrillation up to 3 times at 200 J, 200 to 300 J, 360 J, or biphasic defibrillator at approximately 150 J b.synchronized cardioversion c.epinephrine 10 mL of 1:10 000 solution IV bolus d.atropine 1 mg IV

4

17-For which of the following patients with PEA is sodium bicarbonate therapy (1 mEq/kg) most likely to be most effective? a.the patient with hypercarbic acidosis and tension pneumothorax treated with decompression b.the patient with a brief arrest interval c.the patient with documented severe hyperkalemia d.the patient with documented severe hypokalemia 18Which of the following potential causes of prehospital asystole is most likely to respond to immediate treatment? a. prolonged cardiac arrest b.prolonged submersion in warm water c.drug overdose d.blunt multisystem trauma 19-Which of the following is the correct initial drug and dose for treatment of asystole? a.Epinephrine 2 mg IV b.Atropine 0.5 mg IV c.Lidocaine 1 mg/kg IV d.Epinephrine 1 mg IV 20-Paramedics have arrived with an asystolic 42-year man who was found unconscious, breathless, and pulseless in the hallway of his apartment. CPR is ongoing. The patient is intubated. He has bilateral breath sounds and equal and adequate chest expansion. IV access has been successfully established, with fluid infusion at a “keep open” rate. The patient’s vital signs are follows: PaO2 = 85 mm Hg; Paco2 = 32 mm PH; 7.3; serum potassium = 4.5 mEq/L; core body temperature = 37º C. In this scenario, which is the most likely reversible cause to consider before stopping the resuscitation attempt? a.Tracheal tube in the esophagus b.Drug overdose c.Tension pneumothorax d.Hypothermia 21-Which of the following should be checked as part of the flat line protocol to confirm the presence of asystole and rule out operator or monitoring error as the reason for the isoelectric ECG? a. Check power switch, all connections between the monitor and patient, monitor/defibrillator battery, sensitivity or gain, and lead choice b. Obtain 12-led ECG b. Press the SYNCHRONIZE button on the cardi overter/defibrillator c. Administer a trail defibrillatory shock to rule out occult VF

5

22- You are working in the ED when a 34-year-old woman arrives with a complaint of palpitations. She has a history of mitral valve prolapse and a heart rate of 165 bpm, respiratory rate of 14 breaths/min, and blood pressure of 118/82 mm Hg. Her lungs are clear to auscultation, and she has no hepatomegaly. She denies having any shortness of breath. She is placed on an ECG monitor, which indicates that s narrow-complex, regular tachycardia is present. Which of the following phrases best characterizes this patient’s condition? a. Stable tachycardia b. Unstable tachycardia c. Heart rate appropriate for clinical condition d. Tachycardia with poor cardiovascular function 23- You are working in a clinic when a 58-year-old man walks in complaining of chest pain. He is diaphoretic and complaints of dizziness. He sits in a chair at the triage desk while you check his pulse, which is rapid. As you prepare to attach a cardiac monitor to the patient, the man suddenly slumps over unresponsive. Which of the following best describe his condition? a. stable tachycardia b. unstable tachycardia, possible cardiac arrest c. heart rate appropriate for clinical condition d. tachycardia with adequate cardiovascular function 24- For which of the following patients would immediate cardioversion be indicated? a. a 62-year-old man with rheumatic and aortic valve disease, an irregularly irregular heart rate of 153 bpm, and a blood pressure of 88/70 mm Hg b. a 78-year-old woman with fever, pneumonia, mild chronic congestive heart failure, and sinus tachycardia of 133 bpm c. a 55-year-old man with multifocal atrial tachycardia, a respiratory rate of 12 breaths/min, and a blood pressure of 134/86 mm Hg d. a 69-year-old with a history of coronary artery disease who presents with chest pain, a heart rate of 118 bpm, and ST-segment elevation 25- Which of the following groups of signs would not be consistent with evidence of unstable tachycardia? a. heart rate of 140 bpm, tachypnea, wheezing, and pneumonia in a patient receiving albuterol b. heart rate of 140 bpm with rapid atrial in a patient with aortic stenosis and AMI, blood pressure of 90/55 mm Hg, faint peripheral pulses, disphoresis, tachypnea and rales c. VT in a man complaining of chest pain, shortness of breath, and palpitations d. Heart of 155bpm in a 55-year-old woman with severe chest pain, difficulty breathing, extreme weakness and dizziness, & blood pressure of 88/54 mm Hg

6

26- A 75-year-old man presents to the ED complaining of having lightheadedness and palpitations for 1 week. His heart rate is 160 bpm and irregular; his blood pressure is 100/70 mm Hg. The physical examination is normal, with no evidence of cardiac or circulatory failure. The 12-lead ECG shows rapid atrial fibrillation but is otherwise normal. Which of the following should be included in your initial orders for this patient? a. Oxygen-IV-monitor b. Immediate defibrillation c. No therapy is indicated d. Epinephrine 1 mg IV every 3 to 5 minute 27- A 55-year-old man with known heart failure develops sustained widecomplex tachycardia after an episode of chest pain relieved by nitroglycerin. Currently HR= 150 bpm, Bp= 100/60 mm HG; ECG before the tachycardia = old left bundle branch block, which prevents determination of the widecomplex tachycardia as ventricular or supraventricular in origin. Which of the following is the most appropriate initial medication? a. IV lidocaine b. IV adenosine c. IV amiodarone d. IV verapamil 28- You decide to convert an unstable, symptomatic tachycardia. You place the cardiovereter/defibrillator in synchronization mode and administer as sedative and an analgesic to the patient. Suddenly the patient becomes unresponsive and pulseless, and the ECG rhythm becomes highly irregular, resembling VF. When you attempt to deliver the shock, nothing happens: no shock is delivered and there is no energy transfer. What is the explanation for failure to deliver a shock? a. The defibrillator/cardioverter battery has failed b. The SYNC switch is not functioning properly c. The patient has developed VF and the defibrillator will not deliver a charge because it is attempting to synchronize shock delivery with an R wave d. The monitor cannot synchronize the cardioversion shock because a lead has come loose 29- A 25-year-old woman presents to the ED saying “I’m having another episode of PSVT!” Her prior medical history includes an electrophysiologic stimulation study that confirmed a reentry tachycardia, no WPW, and no pre-excitation. Her heart rate is 180 bpm; she reports palpitations and mild shortness of breath. Vagal maneuvers with carotid sinus massage have no effect on heart rate or rhythm. Which would be the most appropriate next intervention? a. DC cardioversion b. IV dilitiazem c. IV propranolol d. IV adenosine

7

30) You have just evaluated a 60-year-old woman with known Wolff-Parkinson- White syndrome. Her chief complaints is palpitations and mild chest discomfort that started 1 hour ago. Her ECG shows rapid atrial fibrillation at a rate of 175 bpm. Which of the following drugs is contraindicated? a. IV diltiazem b. IV propranolol c. IV digoxin d. All of the above 31)An elderly male patient complains of chest tightness, palpitations, and dizziness. His heart rate is 170 bpm; his blood pressure is 90/60 mm Hg. The ECG shows multifocal atrial tachycardia. Which of the following treatments would be inappropriate? a. DC cardioversion b. IV metopolol c. IV diltiaze d. IV amiodarone 32)A 66-year-old homeless man with a history of chronic alcoholism presents with polymorphic tachycardia. He is tolerating the tachycardia well. You correctly diagnose torsades de poiuntes; HR = 160 bpm; BP = 90/60 mm Hg. On physical examination you find a malnourished man with no evidence of heart failure. Which of the following treatments would be most appropriate at this time? a. Amiodarone b. IV magnesium c. IV lidocaine d. IV procainamide 33)Which of the following conditions can mimic the signs and symptoms of an acute stroke? a. Hypoglycemia b. Cardiac arrest c. Pneumothorax d. Wolff-Parkinson-White syndrome 34)The following patients were given a diagnosis of an acute ischemic stroke. Which of the patients as described has no apparent contraindications to IV fibrinolytic therapy? a. an 80-year-old man presenting within 4 hours of onset of symptoms b. a 65-year-old woman who lives alone and was found unresponsive by a relative c. a 54-year-old man presenting within 3 hours of onset of symptoms d. a 40-year-old woman diagnosed with bleeding ulcers 2 weeks before onset of stroke symptoms 35)A 56-year-old woman arrives at the ED with new onset of facial droop when she smiles, arm drift when she holds both arms out, and inability to speak clearly. Before beginning fibrinolytic therapy, the most important question you need to answer is a. have her vital signs remained stable? b. when exactly did the neurologic signs begin? c. does she have a history of heart attack? d. does she have any medication allergies?

8

36) You are walking through a shopping mall when you encounter a 65-year-old woman who stumbled and fell as she walked out of a store. She complains of a severe headache, has a facial droop, and slurs her words. She also complains of numbness in her right arm and leg. She has difficulty raising her right arm, although her left arm moves freely. When you ask if she takes medications, she says she has “high blood pressure”. Which of the following actions would be most appropriate to take at this time? a. phone EMS immediately and tell the dispatcher that you are with a conscious woman who may be demonstrating signs of a stroke. b. suggest that the woman sit down for a few minutes and see if the symptoms disappear c. offer to drive the woman to the ED of the local hospital suggest that the woman contact her physician immediately 37)Which of the following statements about the use of magnesium in cardiac arrest is most accurate? a.Magnesium is indicated in VF/pulseless VT associated with torsades de pointes b. Magnesium is indicated for shock-refractory monomorphic VT c.Magnesium is indicated in VT associated with a normal QT interval d.Magnesium is indicated for VF refractory to shock and amiodarone or lidocaine 38)A patient is in pulseless ventricular tachycardia.Two shocks and one dose of Epinephrine have been given.The next drug/dose to anticipate to administer is a.Vasopressin 40U b.Amiodarone 150mg c.Amoidarone 300mg d.Lidocaine 0.5mg/kg e.Epinephrine 3m 39)A patient with possible ACS and bradycardia of 42 per minute has ongoing chest Discomfort .What in the initial dose of atropine? a.Atropine 1 mg b.Atropine 0.5mg c.Atropine 0.1mg d.Atropine 3 mg 40) Routine use of vasopressors during management of pulseless VT, VF,or asystole may lead to: a. Improvement of survival to hospital discharge. b. Improvement of long term survival c. No outcome benefit of any type d. Improvement of initial resuscitation with resumption of spontaneous circulation.

9

View more...

Comments

Copyright ©2017 KUPDF Inc.
SUPPORT KUPDF